K
Khách

Hãy nhập câu hỏi của bạn vào đây, nếu là tài khoản VIP, bạn sẽ được ưu tiên trả lời.

29 tháng 4 2019

help me

NV
16 tháng 7 2020

Gọi 3 số đó là a;b;c. Do vai trò của a;b;c là như nhau, không mất tính tổng quát, giả sử \(a\ge b\ge c\)

Từ giả thiết ta có: \(\left\{{}\begin{matrix}abc=1\\a+b+c>\frac{1}{a}+\frac{1}{b}+\frac{1}{c}\end{matrix}\right.\) \(\Rightarrow\) a;b;c không thể đồng thời bằng 1 (vi phạm giả thiết thứ 2)

Nếu a;b;c đều nhỏ hơn 1 \(\Rightarrow abc< 1\) (trái giả thiết)

Nếu a;b;c đều lớn hơn 1 \(\Rightarrow abc>1\) (trái giả thiết)

\(\Rightarrow\) Chỉ có 1 hoặc 2 số trong 3 số lớn hơn 1

Giả sử có 2 số lớn hơn 1 \(\Rightarrow a;b>1\)

Từ giả thiết thứ 2: \(a+b+c>\frac{1}{a}+\frac{1}{b}+\frac{1}{c}\)

\(\Leftrightarrow a+b+\frac{1}{ab}>\frac{1}{a}+\frac{1}{b}+ab\)

\(\Leftrightarrow a+b+\frac{1}{ab}>\frac{a+b}{ab}+ab\)

\(\Leftrightarrow a+b-\frac{a+b}{ab}+\frac{1}{ab}-ab>0\)

\(\Leftrightarrow\left(a+b\right)\left(\frac{ab-1}{ab}\right)-\frac{\left(ab-1\right)\left(ab+1\right)}{ab}>0\)

\(\Leftrightarrow\left(ab-1\right)\left(\frac{a+b}{ab}-\frac{ab+1}{ab}\right)>0\)

\(\Leftrightarrow a+b-ab-1>0\)

\(\Leftrightarrow\left(a-1\right)\left(1-b\right)>0\) (vô lý do \(\left\{{}\begin{matrix}a>1\\b>1\end{matrix}\right.\) \(\Rightarrow\left(a-1\right)\left(1-b\right)< 0\))

Vậy điều giả sử là sai

Hay trong 3 số có đúng 1 số lớn hơn 1

17 tháng 7 2020

mình cảm ơn

27 tháng 4 2015

Đặt A = p + p +2 = 2p +2 = 2(p +1)

p +2 = p -1 +3

Xét 3 số liên tiếp : p -1 , p , p +1 có 1 và chỉ 1 số chia hết cho 3

Vì p nguyên tố lớn hơn 3 nên p không chia hết cho 3. Mặt khác p -1 không chia hết cho 3, vì nếu chia hết cho 3 thì p +2 chia hết cho 3, trái với gt là p +2 là số nguyên tố >3. Vậy chỉ còn p+1 chia hết cho 3 => 2(p +1) chia hết cho 3 tức A chia hết cho 3 (*)

Ta lại có p nguyên tố >3 nên p là số lẻ => p = 2k +1 => A = 4k + 4 chia hết cho 4 (**)

mà (3,4) =1 (***)

Từ (*) , (**), (***) => A chia hết cho 12

6 tháng 1 2018

toi có cach khac

20 tháng 5 2016

1+2+3=6 ma 6 khong phai la so nguyen to va 6>3